If MrBeast bought 69 Lamborghini's and bought 420 Rolex watches how many items in all would he have bought altogether?

Answers

Answer 1
it’ll be 489 altogether

have a great day :)
Answer 2

Answer: 489

Step-by-step explanation:

420+69=489


Related Questions

WILL MARK BRAINILIEST
are the angles corresponding, alternate interior, alternate exterior, or constructive interior​

Answers

Answer:

maybe it is corresponding

Please help me ASAP and i mean ASAP

Answers

Answer:

no

Step-by-step explanation:

no

a nuber to be multiplied is called

Answers

Answer:

7×5 4×5 777⁷7777777777×7

Answer:

it is called factors

Step-by-step explanation:

Sean and sarah spend $70 a month eating out how much money would they have after 15 years if they invested the money instead at 4 percent compounded monthly round your answer to the nearest hundredth

Answers

Answer:

Future Value= $17,170.12

Step-by-step explanation:

Giving the following information:

Monthly deposit (A)= $70

Interest rate (i)= 0.04/12= 0.0033

Number of periods (n)= 15*12= 180 months

To calculate the future value (amount saved), we need to use the following formula:

FV= {A*[(1+i)^n-1]}/i

A= monthly deposit

FV= {70*[(1.0033^180) - 1]} / 0.0033

FV= $17,170.12

I NEED HELP ASAP IM STRESSJNG OUT

Answers

Answer:

A

Step-by-step explanation:

Using the Sine rule in Δ ABC

∠ B = 180° - (30 + 50)° = 180° - 80° = 100° ( sum of angles in triangle ) , then

[tex]\frac{13}{sin100}[/tex] = [tex]\frac{BC}{sin30}[/tex] ( cross- multiply )

BC × sin100° = 13 × sin30° ( divide both sides by sin100° )

BC = [tex]\frac{13sin30}{sin100}[/tex] ≈ 6.6 ( to the nearest tenth )

Given the following system of equations, what value of x makes the equations true?
x+y= 3.25
x-y= 1.75

Answers

x=2.5!! hope this helps
Answer: x=2.5 , y=0.75

Explanation:

x+y=3.25 => y=3.25-x ——(1)

x-y=1.75 ——(2)

Substitute equation (1) in (2) then,

x-(3.25-x)=1.75

x-3.25+x=1.75

2x-3.25=1.75

2x=1.75+3.25

2x=5

x=5/2=2.5

Substitute vale of x in equation (1) we get

y=3.25-2.5

y=0.75

Therefore, x=2.5 , y=0.75

f(x) = x^2. What is g(x)?

Answers

Answer:

This becomes way easier if you have the knowledge of inverse functions.

Let me make this clear. You have f(x)=x2+1 f(x)=x2+1 and g(f(x))=1/(x2+4)g(f(x))=1/(x2+4). Now pause and think about the second function. The function is defined as g(f(x))g(f(x)), right. now what if there is some way that you could manipulate this function and some how change it to g(x)g(x). Or think about with what other function should you multiply f(x)f(x) [the function in g(f(x))g(f(x)) ] to get xx [so that the function would be g(x)g(x)]. It is f(x)−1f(x)−1 right.

So let us find f(x)f(x) inverse. f(x)=x2+1f(x)=x2+1 then f(x)−1=x−1−−−−−√f(x)−1=x−1 right [ just swipe the xx with f(x)

write this expression using numbers and symbols two less then six times a number
a. 2n-6
b.2n+6
c.6n-2
d.6-2​

Answers

Answer:

d

Step-by-step explanation:

becaus ei did

What is a solution of -8x+5>=11
A.-1/2
B.3
C.0.25
D.-1

Answers

Answer:

i got -2

Step-by-step explanation:

1 step; 8*x-5-(11)=0

2 step; 8x - 16  =   8 • (x - 2)

3 step;  Solve :    8   =  0

4 step; Solve  :    x-2 = 0

final part  Add  2  to both sides of the equation :  

                     x = 2

The slope is ____ and the y-intercept is ___?​

Answers

Answer:

The slope is -1 and the y-intercept is -1 ?

[tex]slope = \frac{ - 6 - 1}{5 - ( - 2)} \\ = \frac{ - 7}{7} \\ = - 1 \\ y - intercept : \\ y = mx + c \\ 1 = ( - 1 \times - 2) + c \\ c = - 1 \\ [/tex]

Answer:

slope -1 and y intercept is -1

hope this helps

have a good day :)

Step-by-step explanation:

Given the linear equation 5x+3y-9=0. Write another linear equation in two variables such that the geometrical representation of the pair so formed are parallel lines​

Answers

Answer:[tex]5x+3y+1=0[/tex]

Step-by-step explanation:

Given

The linear equation is [tex]5x+3y-9=0[/tex]

The other linear equation parallel to the given line can be represented by

[tex]5x+3y+c=0[/tex]

Here, c can take any value . for example c=1, line becomes

[tex]\Rightarrow 5x+3y+1=0[/tex]

How many solutions does the system have?

4x−6y=−24
2x−3y=−6

Answers

Answer:

none

Step-by-step explanation:

if we plot it, it's clear, but you might also spot that the plotted lines will be parallel in the equation

because x and y in line two are the same multiple of these in line one (both values are just the half)

plotted with desmoes

Share $150 among John, Mary, and Sarah so that each of the two girls receives twice as much as John. Calculate the amount of money Jon received.

Answers

They get 60 dollars each Sarah and Mary and John gets 30 dollars because 60+60=120 and add 30 and it is 150 and 30 is half of 60

Find the value of x.

Answers

Answer:

x = 82°

Step-by-step explanation:

The opposite angles of a cyclic quadrilateral are supplementary, sum to 180°

x + 98° = 180° ( subtract 98° from both sides )

x = 82°

To make a profit, a company’s revenue must be greater than its operating costs. The company’s revenue is modeled by the expression 7.5x – 100, where x represents the number of items sold. The company’s operation costs are modeled by the expression 79.86 + 5.8x. How many items does the company need to sell to make a profit?

Answers

Answer:

106 items

Step-by-step explanation:

We are actually finding the "break even" point, where revenue = costs.

Set the revenue 7.5x - 100 equal to the costs 79.86 + 5.8x:

7.5x - 100 = 79.86 + 5.8x

Group the x terms together:    

1.7x  - 100  =  79.86

and then group the constants together on the right side:

  1.7x = 179.86

Solving for x, we get 105.8 (items)

Round this up:  106 items

The company will begin making a profit as production passes 106 items.

Answer:

first one : 7.5x -100 > 79.86 + 5.8x

second one : x > 105.8

last one : 106

Step-by-step explanation:

Identify the slope and y-intercept for the line y = -2x -3.

Answers

Answer:

The slope is -2 and the y intercept is -3

Step-by-step explanation:

y = -2x -3

This equation is written in slope intercept form

y = mx+b where m is the slope and b is the y intercept

The slope is -2 and the y intercept is -3

Use the tangent ration to find the height of the tree. Round to next whole number.

Answers

Answer:

74.8925767 rounded to 75

Step-by-step explanation:

Find the volume of a sphere with the given dimensions. radius is 4cm

Answers

Answer:

Volume of a sphere- (4/3) x π x r3

(4/3) x 3.14 x 4 x 4 x 4

= (4/3) x 3.14 x 64

= 267.94

someone help me answer these two​

Answers

1. Domain (6, 3) to (11, 2)
Range (9,0) to (6,3)
Not a function because two inputs cannot share the same output

2. Domain (1, 50) to (5, 200)
Range (1,50) to (5, 200)
Yes it is a function

Which function graph is shown

Answers

Answer:

A) y=3cos(-x)

Step-by-step explanation:

Because the amplitude of the graph is 3, this eliminates options C and D as their amplitudes are different. With the period of the graph being 2π, B cannot be correct because that would have a period of π/2. Therefore, A is correct.

✨Please help me :) ✨

Answers

Answer:

Jaden would have to save $25 each week for next 6 weeks to get the $290 bike.

Step-by-step explanation:

What do we know:

He has $140 to start withHe saves $25 each weekHis bike costs $290

What are we solving for;

We are solving for x (number of weeks it would take Jaden to buy a $290 bike), when y = 290

Equation would be:

y = 25x + 140

So for Jaden to buy a $290 bike, y would have to be replaced with 290, and solve for x;

y = 25x + 140

=> 290 = 25x + 140

=> 290 - 140 = 25x

=> 150 = 25x

=> 150 / 25 = x

=> 6 = x

Therefore, Jaden would have to save $25 each week for next 6 weeks to get the $290 bike.

Hope this help!

10 points!
If you are going to answer, actually answer!

Answers

Answer:

1/27

Explanation:-

(1/3)^3=(1x1x1)/(3x3x3)=1/27

Answer:

0.03703703703 or

                                                                                                                                 ²/27

sorry i am late but still hope I helped

Step-by-step explanation:

the simplified form of 0.02(0.06+0.008)is​

Answers

Answer:

Step-by-step explanation:

Use PEDMAS. So first addition has to be done as it is inside parenthesis. After that multiplication has to be done

0.060  +

0.008

0.068

In decimal multiplication, first multiply the numbers ie, 2 * 68 which is 136. Then count the number of decimal places in both the multiplicands and keep the same number of decimal places in the result(136).

0.02 * 0.068 = 0.00136

Answer:

0.00136

Step-by-step explanation:

According to Order of Operations rules, we must do work that appears inside parentheses first.

Therefore we combine 0.06 and 0.008 to obtain 0.068.

Next comes multiplication:  0.02(0.068) = 0.00136

I really need help with c,d and e

Answers

Answer:

Step-by-step explanation:

By using regression line calculator,

Equation of the line of best fit,

c). y = 0.74x + 0.03

Here, x = Number of pints

y = Weight in pounds

d). We have to find the weight of 10 pints of the blueberries,

By substituting x = 10 in the equation,

y = 0.74(10) + 0.03

y = 7.4 + 0.03

y = 7.43 pounds

e). If per pound cost of the blueberries = $2.25

By substituting y = 2.25 in the equation,

2.25 = 0.74x + 0.03

x = [tex]\frac{2.22}{0.74}[/tex]

x = $3.00 per pound

Therefore, cost of 10 pounds blueberries = 3 × 10

                                                                     = $30

help me solve for m<4 and m<5 and x

Answers

Answer:

x = 18

Angle 4 = 31 degree

Angle 5 = 59 degree

Step-by-step explanation:

According to the question, angle 3 is 90 degree and the sum of angle 4 and 5 is equal to the angle 3 as they are vertically opposite angles.

Angle 4 + angle 5 = 90 degree

Angle 4 = 2 x - 5

angle 5 = 4 x - 13

So, according to the question,

2 x - 5 + 4 x - 13 = 90

6 x - 18 = 90

6 x = 108

x = 18

So, angle 4 = 2 (18) - 5 = 31 degree

So, angle 5 = 4 (18) - 13 = 59 degree

Pls help me solve pls show how you got the answer

Answers

Answer:

48 length units

Step-by-step explanation:

area of each squre = 9

side length of each square = sqrt(9) = 3

Length of each side of grid = 4*3 = 12

Perimeter of grid = 4 sides = 4*12 = 48 length units

ANSWER:

48length units

hope this helps

Over 5 straight plays, a football team gained 8 yards, lost 4 yards, gained 7 yards, gained 3 yards, and lost 11 yards. What is the team's position no compared to their starting position? WILL GIVE BRAINLIEST ALSO BE CORRECT PLS THX

Answers

for this one, i’ll explain. set it up so it looks like this : 8-4+7+3-11, and then simplify.

The following graphs have no scales assigned to them. Which of them could
not possibly be density curves for a continuous random variable if they were
provided with the right scale?
Check all that apply.

A. Graph A
B. Graph B
C. Graph C
D. Graph D
E. Graph E

Answers

The graph A , graph C, could not possibly be density curves for a continuous random variable is they were provided with the right scale.

Given four graphs of different nature as options.

We are required to find the graph that is not continuous variable.

Continuous lines are those lines which when decreases, decreases with continuity and when increases,increases with continuity.

When we observe the graph A,graph C. In these graphs the value of function keeps increasing and decreasing a lot and not in continuity.

Hence graph A, graph C could not possibly be density curves for a continuous random variable is they were provided with the right scale.

Learn more about continuity at https://brainly.com/question/24637240

#SPJ1

Answer:

The answer is A, D, and E.

Step-by-step explanation:

Convert 5 km 36 m into km

Answers

Answer:

5.036 km

Step-by-step explanation:

36  m ÷ 1000  = 0.036 km

5 km + 0.036 km = 5.036 km

That’s the answer becoz it correct

find hcf and LCM of 40 ,60 and 80​

Answers

hear here is your answer in attachment attach

Answer:

Step-by-step explanation:

40 = 2*2*2*5  

60 = 2 * 2 * 3 * 5

80= 2 * 2 * 2 * 2 * 5

LCM = 2 * 2 * 2 * 2 * 5 * 3 = 240

HCF = 2*2*5= 20

Other Questions
PLEASE HELP ITS TIMED LITTERALY IM BEGGING :(( 32 points WITH BRAINLIEST!! how to write an article steps involve Rafael va de compras para algunas personas. Mirael model escribe las oraciones con los pronombre correcto sustituyendo los nombres desalare Please help Ill give brainliest ima give branliest if you answer this correctly 1.What symbolism are used in the cartoon?2.What is the message in the cartoon?3.Do you think this scenario is true in the Philippines? Why or Why not? can u help me plzx part 2 During June, the company purchased 160,000 pounds of direct material at a total cost of $1,056,000. The company manufactured 20,000 units of product during June using 120,800 pounds of direct materials. The price variance for the direct materials acquired by the company during June is: (Do not round intermediate calculations.) summary on scaler quantity Please, I'm desperate need of help I will mark Brainliest to the one who answers with the full process! Gold will Bless You! Two angles of a triangle are 85 and 65, and the longest side of the triangle is 34 cm. How long is the shortest side? Include a labelled diagram. Yoo need help ? Can anyone help me or lead me to where I can get good micro help give two major differences between prokaryotes and eukaryotes Megan is shopping at a store that sells jewelry, scarves, and purses. The cost of all of the items at the store includes tax. Megan buys 3 bracelets and 3 necklaces. Each bracelet costs $5. Megan pays the clerk $40 and gets $4 change. What is the cost, in dollars, of one necklace? I would appreciate if someone could answer this Which expression is equivalent to 5^15 x 5^5 Select the correct answer.Which expression is equivalent to the glven expression?(3m-4)^3(3m^5)A. 81/m^2 B.27/m^7 C. 27/m^2 D.81/m^7 Suppose that QRS is isosceles with base q r. Suppose also that m q equals (5x - 14(degrees and Mr equals (4x - 2)degreePLEASE HELP ME The following data are available for Sampson Corporation.Net income$200,000Depreciation expense60,000Dividends paid90,000Loss on sale of land15,000Decrease in accounts receivable30,000Decrease in accounts payable45,000Net cash provided by operating activities is:A. $160,000.B. $140,000.C. $260,000.D. $240,000. Which of these statements is a paradox ?A. Blood is thicker than water B. You have to hurt him to help himC. I had to tap into my inner strength D. All things are beautiful in their own way Simplify (2m^4)^4*-n^5this is the last questions but I'm confused